What is Angular momentum: Definition and 1000 Discussions

In physics, angular momentum (rarely, moment of momentum or rotational momentum) is the rotational equivalent of linear momentum. It is an important quantity in physics because it is a conserved quantity—the total angular momentum of a closed system remains constant.
In three dimensions, the angular momentum for a point particle is a pseudovector r × p, the cross product of the particle's position vector r (relative to some origin) and its momentum vector; the latter is p = mv in Newtonian mechanics. Unlike momentum, angular momentum depends on where the origin is chosen, since the particle's position is measured from it.
Just as for angular velocity, there are two special types of angular momentum of an object: the spin angular momentum is the angular momentum about the object's centre of mass, while the orbital angular momentum is the angular momentum about a chosen center of rotation. The total angular momentum is the sum of the spin and orbital angular momenta. The orbital angular momentum vector of a point particle is always parallel and directly proportional to its orbital angular velocity vector ω, where the constant of proportionality depends on both the mass of the particle and its distance from origin. The spin angular momentum vector of a rigid body is proportional but not always parallel to the spin angular velocity vector Ω, making the constant of proportionality a second-rank tensor rather than a scalar.
Angular momentum is an extensive quantity; i.e. the total angular momentum of any composite system is the sum of the angular momenta of its constituent parts. For a continuous rigid body or a fluid the total angular momentum is the volume integral of angular momentum density (i.e. angular momentum per unit volume in the limit as volume shrinks to zero) over the entire body.
Torque can be defined as the rate of change of angular momentum, analogous to force. The net external torque on any system is always equal to the total torque on the system; in other words, the sum of all internal torques of any system is always 0 (this is the rotational analogue of Newton's Third Law). Therefore, for a closed system (where there is no net external torque), the total torque on the system must be 0, which means that the total angular momentum of the system is constant. The conservation of angular momentum helps explain many observed phenomena, for example the increase in rotational speed of a spinning figure skater as the skater's arms are contracted, the high rotational rates of neutron stars, the Coriolis effect, and the precession of gyroscopes. In general, conservation limits the possible motion of a system but does not uniquely determine it.
In quantum mechanics, angular momentum (like other quantities) is expressed as an operator, and its one-dimensional projections have quantized eigenvalues. Angular momentum is subject to the Heisenberg uncertainty principle, implying that at any time, only one projection (also called "component") can be measured with definite precision; the other two then remain uncertain. Because of this, the axis of rotation of a quantum particle is undefined. Quantum particles do possess a type of non-orbital angular momentum called "spin", but this angular momentum does not correspond to a spinning motion.

View More On Wikipedia.org
  1. K

    Eigenfunctions of the angular momentum operator

    Hi everyone, I tried to find the Eigenstate of the angular momentum operator myself, more specifically I tried to find a Function Y_{lm}(\theta,\phi) with L_zY_{lm}=mħY_{lm} and L^2Y_{lm}=l(l+1)ħ^2Y_{lm} where L_z=-iħ\frac{\partial}{\partial \phi} and...
  2. H

    Moment of Inertia and Bike stability

    So is a Bike driver stable when the bike is running because the bike wheels has a certain moment of inertia about the horizontal axis ,which might alter(mi gets lesser) if the direction of the axis changes ? Thanks in advance
  3. Chrono G. Xay

    Calculate the 'Feel' of a Drumhead?

    As another of my personal music projects, I have wondered if it would not be possible to calculate the 'feel' of a drumhead (i.e. the amount of 'give' expressed as transverse displacement 'z' that an equally pre-tensioned circular membrane of radius 'r' experiences when struck on its plane at a...
  4. K

    Angular momentum about inconvenient pivot point

    A rod of length L and mass M is rotating with angular velocity w about its center of mass. What is its angular momentum with respect to a rotating point 1/4L from the rod's end? What about a stationary point? Homework Equations With the respect to the center of mass, angular momentum L = Iw =...
  5. B

    Quantum angular momentum proportionality constant

    Hi, not a homework problem as such, but I am studying introductory quantum physics and having some trouble understanding how applying the rising or lowering operator for angular momentum implies: L+/-|l,m> ∝ |l, m+/-1> Basically, my question is the same as the first part (Q1) as described in...
  6. Titan97

    Conservation of angular momentum

    Homework Statement Two discs of different radii and masses are kept on a smooth horizontal table and both are free to rotate about their fixed central vertical axis. One of them is given some angular velocity while other is stationary. The rims are brought in contact. There is friction between...
  7. D

    Understanding Angular Momentum and Its Effects on Spinning Objects

    I'm trying to find out if this is angular momentum or something else. It says this, "If an object spins in a certain direction any pieces that break off that object must also spin in the same direction." Is that angular momentum or something else?
  8. G

    Law of the lever: Conservation of energy or angular momentum

    Hi, Some "derivations" of the law of the lever argue with conservation of energy: If one arm of the lever of length ##r_1## is pulled by a distance ##s_1## with force ##F_1##, the other arm moves by a distance ##s_2=s_1 \frac{r_2}{r_1}##. From conservation of energy ##E=F_1 s_1=F_2 s_2## it...
  9. M

    Conservation of angular momentum invariance

    Homework Statement Given a reference frame O' moving at a constant speed $\vec{V}$ in relation to another reference frame O, I want to prove that ##\vec{r_{1B}} \times m_1\vec{v_{1B}} + \vec{r_{2B}} \times m_2\vec{v_{2B}} = \vec{r_{1F}} \times m_1\vec{v_{1F}} + \vec{r_{2F}} \times...
  10. M

    Total angular momentum of EM fields

    Homework Statement The problem deals with a charged (Q) rotating sphere around its axis (Ω_0) z^^ (z hat) of radius a. I'm asked to find the total angular momentum of the EM fields. 2. The attempt at a solution There is a solution posted to this question and I was just wondering why my...
  11. Prashan Shan

    Angular momentum of a half filled waterbottle....

    Will the angular momentum of a half filled waterbottle increase? or decrease? or remain the same?
  12. D

    Finding the Total Angular Momentum

    Say, I have two spin-1/2 particles in the states characterized by ##(n=2, l=1, m_l=1, m_s=1/2)##and##(n=2, l=1, m_l=1, m_s=-1/2)##. Now, using something like the jj coupling scheme, I first find out the (orbital+spin)angular momentum for the individual particles:(i) ##| 11\rangle...
  13. C

    Orbital and Spin Angular momentum of light derivation

    Homework Statement The momentum carried by an electromagnetic field is [;\vec{P}(\vec{x}, t) = \frac{1}{4\pi c} \int d\vec{x}\vec{E}(\vec{x},t) \times \vec{B}(\vec{x},t);] show that for a finite field extension [;\vec{J}(\vec{x}, t) = \frac{1}{4\pi c}\int -i...
  14. R

    Doubts on angular momentum [master's degree exam]

    Two hollow spheres, both the mass and radius R M , which are rotating around a center of mass ( CM ) , with an initial period To, are kept distant from each other by an ideal wire with a distance of 8R. At a given instant a motor is driven by wrapping the wire and making the two spheres meet...
  15. B

    What Are the Possible Values of L_x in a Quantum System with l=1?

    Homework Statement I've a physical system with ##l=1## and I have to calculate the values I can obtain if I measure ##L_x## and their probability.Homework Equations I know that: - the values I can obtain are ##\ m=0, \pm 1## - ##\displaystyle L_x=\frac{L_+ + L_-}{2}## - ##L_x|1, m>_x=\hbar m...
  16. Thom_Silva

    A Rotating motor compressing a spring

    Hello, I've recently came across this video (), where the authors use a motor to compress springs and therefore achieve locomotion. I've been thinking why is there a resulting downward net force. But i can't really figure it out. Thank you for time :) See the video from 1.16 minutes and...
  17. Z

    Angular Momentum in a pulley-block system

    Homework Statement A counterweight of mass m 5 4.00 kg is attached to a light cord that is wound around a pulley. The pulley is a thin hoop of radius R = 8.00 cm and mass M = 2.00 kg. The spokes have negligible mass. When the counterweight has a speed v, the pulley has an angular speed v = v/R...
  18. D

    Cylinder with point mass angular momentum

    Homework Statement A uniform cylinder of mass M and radius R can be rotated about a perpendicular axle through its centre. A particle of mass m is attached to the cylinder's rim. The system is rotated with angular velocity w about the axle, which is held in a fixed direction during the motion...
  19. vinicius0197

    Deducing Kepler's second law from Newton's laws?

    I've searched a little bit and found that I can derive kepler's third law from Newton's law of gravitation. That's okay. But I want to deduce kepler's second law too: "An imaginary line joining a planet and the sun sweeps out an equal area of space in equal amounts of time". I know it's possible...
  20. K

    R-parity and conservation of angular momentum

    Assuming R-pairity and thus the creation/destruction of supersymmetric particles happens in pairs, how is angular momentum conserved when a particle and its supersymmetric partner have different spin by 1/2?
  21. H

    How does the velocity of a mass spun around a pencil change?

    Suppose a mass ##m## is attached to the end of a string whose other end is attached to a cylindrical pencil. The mass is then spun around the pencil in a circle (whose centre coincides with the centre of the pencil) such that the string wraps around the outer surface of the pencil, thereby...
  22. 24forChromium

    How Does Angular Impulse Affect a Rotating Cylinder's Momentum?

    Hopefully the image is self-explanatory, if not: A cylinder is rotating around its central axis with angular momentum L1; an angular impulse, ΔL is then added to the cylinder, perpendicularly with respect to L1. The hypothetical result is: the cylinder has one angular mometum at the end...
  23. G

    Canonical definition of Angular Momentum,

    Let's start with an arbitrary solid body rotating around a fixed axis of rotation with angular velocity ##\vec \omega## in the ## \hat z## direction. For simplicity, let's say the origin O is on the axis of rotation. Take a look at the picture I sketched in the next post. Tried my best to be...
  24. O

    Physics Experiment Help (Torque, Angular Momentum, etc)

    < Mentor Note -- thread moved to HH from the technical physics forums, so no HH Template is shown > Hey, So I am not sure if this is in the right section but feel free to move it. We are conducting an experiment at school at the moment and are having difficulty understanding all the theory...
  25. M

    A Question Regarding Black Holes

    Hello people, I have a question regarding black holes. The way i understand it, black holes form in supernovas, and they occur because the gravitational pull of the stellar remnant is so great that nothing can stop it, and it basically collapses down to a singe point, virtually nothing... Now...
  26. 24forChromium

    How do you add angular momentum in different dimensions?

    Say a ring is spining around the z-axis, an angular impulse is then applied to it in the x-axis, what is the resultant motion qualitatively and quantitatively? How can it be calculated? (You can make up the quantity of z-angular momentum and x-angular impulse)
  27. S

    Net Angular Momentum of Satellite with Reaction Wheel

    I am modelling the attitude dynamics of a satellite. The satellite has a reaction wheel in 1 plane to help control the attitude. There is significant debate about the equation for the net angular momentum of the satellite and what inertia tensors should be used regarding parallel axis theorems...
  28. G

    Angular Momentum Conservation in Gear Interactions

    Lets say one gear is rotating with some angular velocity and its angular momentum vector is pointing up. A second gear (not rotating) is then allowed to mesh with the first. The second gear has the same radius and moment of inertia as the first. Is not the angular momentum vector of the second...
  29. NanaToru

    Metaphor for orbital angular momentum?

    I've been tutoring for chemistry and someone asked me to clarify the difference of spin angular momentum and orbital angular momentum without math. I was trying to think of a metaphor, but I wanted to make sure it's a fair one--the spin angular momentum is like Earth rotating on its own axis...
  30. D

    Total angular momentum operator for a superposition

    Hi all, Quick quantum question. I understand the total angular momentum operation \hat{L}^2 \psi _{nlm} = \hbar\ell(\ell + 1) \psi _{nlm} which means the total angular momentum is L = \sqrt{\hbar\ell(\ell + 1)} But how about applying this to an arbitrary superposition of eigenstates such as...
  31. L

    What operators are involved in angular momentum states?

    Hoping this is in the right section! The module is nuclear and atomic physics but it crosses over into quantum occasionally. I've attached an image of the bit I'm trying to work out. I've got an exam on this topic in just over a week, so sorry if these posts get annoying, I have a feeling I'm...
  32. rpthomps

    Angular momentum along a sloping line

    Homework Statement A 1.0 kg particle is moving at a constant 3.5 m/s along the line y=0.62x +1.4, where x and y are in meters and where the motion is toward the positive x and y directions. Find its angular momentum about the origin2. Attempt at a...
  33. rpthomps

    How Does Mass and Speed Affect the Circular Motion of a Rod with Attached Putty?

    Homework Statement A rod of length l and mass M is suspended from a pivot, as shown The rod is struck midway alongs its length by a wad of putty of mass m moving horizontally at speed v. The putty sticks to the rod. Find an expression for the minimum speed v, that will result in the rod’s...
  34. J-dizzal

    Angular momentum of ball problem

    Homework Statement In the figure, a 0.400 kg ball is shot directly upward at initial speed 40.4 m/s. What is its angular momentum about P, 6.65 m horizontally from the launch point, when the ball is (a) at maximum height and (b) halfway back to the ground? What is the torque on the ball about P...
  35. Sarah00

    Angular Momentum of Mass: Understanding the Equations and their Application

    Homework Statement Homework Equations L = mvr L = Iw The Attempt at a Solution I did not attend this lesson due to some reasons. I read it from the book but I could not understand it well. I know the linear momentum well. However, in angular momentum, we have two equations I don't know...
  36. B

    Angular Momentum of a Uniform Rod

    << Mentors have notified the OP to show their Attempt at a Solution >> 1. Homework Statement A uniform rod of length L1 = 1.5 m and mass M = 2.8 kg is supported by a hinge at one end and is free to rotate in the vertical plane. The rod is released from rest in the position shown. A particle of...
  37. gracy

    When to apply conservation of angular momentum?

    So my question is when to apply conservation of angular momentum?When there is no external force ,right?But in the case below A mass m moves in a circle on a smooth horizontal plane with velocity v0 at a radius R0. The mass is attached to a string which passes through a smooth hole in the plane...
  38. avito009

    Difference between "Spin" and "Angular Momentum"?

    I am hesitant to write this post. I am a bit overwhelmed that many members have questions of a higher level. They may be reading scientific journals. My question is very basic. I might be called stupid. Coming to the question. Is angular momentum same as spin of an electron?. But I read that...
  39. robbertypob

    Interstellar Movie - Conservation of Angular Momentum

    OK, so I'm right in the middle of watching Interstellar and I've just seen the part where they have docked with the 'mothership' and they fire up their engines to get it to rotate. This is so they can generate a centrifugal force to simulate gravity. My question is - first the ship was not...
  40. kzf

    Kinematics derivation of conservation of angular momentum

    Homework Statement A welding robot consists of an arm (thin rod) that can rotate about the origin point O, and a welding tip, which can freely move along the rod, from the outermost point of the arm A all the way to the center point O. The design invokes two electric motors, one to turn the...
  41. K

    Calculating angular speed of a ball after collision

    Hi, I've been wondering is there anyway of calculating the angular speed of a ball after there is a collision of it and another mass. For example a baseball bat hitting the ball. I have not looked up on angular momentum, but is angular momentum involved in this? Based on common sense, I think...
  42. LarryS

    Orbital Angular Momentum: Need at least 2 particles?

    The definition of orbital angular momentum, whether for classical mechanics or for quantum mechanical operators, is rxp. Technically, according to this definition, one particle can possesses orbital angular momentum - in this case about the origin. But I cannot think of any examples, in...
  43. omega5

    Rings on a rod: angular momentum conceptual question

    Homework Statement A uniform rod of mass 3.15×10−2kg and length 0.380m rotates in a horizontal plane about a fixed axis through its center and perpendicular to the rod. Two small rings, each with mass 0.250kg , are mounted so that they can slide along the rod. They are initially held by catches...
  44. K

    What Determines the Allowed Values of lα in Alpha-Decay?

    Hi, I am reading about alpha-decay. I read that the decay 2- --> 2x requires lα = 1, 2, 3 or 4. Why does it have to be one of these numbers?
  45. K

    Big Bang & Angular Momentum Conservation

    Hi All, Is the Angular momentum conserved at the singularity ? Thanks in advance
  46. G

    Parity and total angular momentum

    Hi, I'd like to know how to calculate parity and total angular momentum of nuclei which have even Z and even N and also Z and N are magic numbers, such as 8O8 or 20Ca20 (the number before the element is Z and the after one is N). I don't know how to insert LaTeX formulas. Thank you in andvance
  47. D

    Confused about angular momentum

    Hi. For the angular momentum l=2 case are the L2 and Lz matrices both 5 x 5 matrices with the following eigenvectors ? ## \begin{pmatrix} 1\\0\\0\\0\\0 \end{pmatrix} ## , ## \begin{pmatrix} 0\\1\\0\\0\\0 \end{pmatrix} ## , ## \begin{pmatrix} 0\\0\\1\\0\\0 \end{pmatrix}##,## \begin{pmatrix}...
  48. blue_leaf77

    Role of Angular Momentum in Defining Vector Operator ##\mathbf{V}##

    A vector operator ##\mathbf{V}## is defined as one satisfying the following property: ## [V_i,J_j] = i\hbar \epsilon_{ijk}V_k## where ##\mathbf{J}## is an angular momentum operator. My question is what is the role of ##\mathbf{J}##, does it have to be the total angular momentum from all...
  49. DavideGenoa

    Derivative of angular momentum

    Hi, friends! Let the quantity ##I\boldsymbol{\omega}## be given, where ##I## is an inertia matrix and ##\boldsymbol{\omega}## a column vector representing angular velocity; ##I\boldsymbol{\omega}## can be the angular momentum of a rigid body rotating around a static point or around its -even...
  50. N

    Angular momentum & Energy using Yukawa's potential

    Hello there! I was doing my Gravitation problems and I found this problem that I'm unable to solve. Yukawa's theory for nuclear forces states that the potential energy corresponding to the attraction force produced by a proton and a neutron is: U(r) = \frac{k}{r}e^{-\alpha r},\ k<0,\ \alpha > 0...
Back
Top